find the value of angle a

Find The Value Of Angle A

Answers

Answer 1

Answer:

angle A = 153 degrees

Step-by-step explanation:

solve for the interior missing angle.

so from the interior we know that we have 2 angles

1 is 63 degrees and 1 is 90 degrees because we see that there is a right angle and all right angles = 90 degrees, we also know that the sum of all angles in a triangle should equal 180 degrees. if we have a 63 degree angle and a 90 degree angle we know that the sum of both of them is 153 degrees we still need to find that missing angle and we can do that by minusing 153 from 180 which is 27 so the measurement of the interior angle of the triangle is 27 but that is not what we are looking for but the answer is already given to us because we know that a straight line is 180 degrees so we just minus that missing angle from 180 degrees we conclude that A = 153 degrees


Related Questions

CAN SOMEONE PLEASE HELP

Answers

[tex]\qquad\qquad\huge\underline{{\sf Answer}}♨[/tex]

Here's the solution ~

[tex]\qquad \sf  \dashrightarrow \: \dfrac{2 {}^{n} - 1}{ {4}^{n} - 1} [/tex]

[tex]\qquad \sf  \dashrightarrow \: \dfrac{2 {}^{n} - 1}{ {( {2}^{2}) }^{n} - 1} [/tex]

[tex]\qquad \sf  \dashrightarrow \: \dfrac{2 {}^{n} - 1}{ {( {2}^{n}) }^{2} - 1} [/tex]

Now we will use this identity in denominator

[ a² - b² = (a + b)(a - b) ]

[tex]\qquad \sf  \dashrightarrow \: \dfrac{ \cancel{(2 {}^{n} - 1)}}{ {( {2}^{n} }^{} + 1) \cancel{( {2}^{n} - 1)}} [/tex]

[tex]\qquad \sf  \dashrightarrow \: \dfrac{ 1}{ {( {2}^{n} }^{} + 1) } [/tex]

Answer:

[tex]\sf \dfrac{1}{2^n+1}[/tex]

Step-by-step explanation:

Given expression:

[tex]\sf \dfrac{2^n-1}{4^n-1}[/tex]

Rewrite [tex]\sf 4^n[/tex]:   [tex]\sf 4^n=(2^2)^n=2^{2n}=(2^n)^2[/tex]

Rewrite 1:  1 = 1²

[tex]\sf \implies \dfrac{2^n-1}{(2^n)^2-1^2}[/tex]

Apply difference of two square formula to the denominator

[tex]\sf a^2-b^2=(a+b)(a-b)[/tex]

[tex]\sf \implies \dfrac{2^n-1}{(2^n+1)(2^n-1)}[/tex]

Cancel out the common factor [tex]\sf 2^n-1[/tex] :

[tex]\sf \implies \dfrac{1}{2^n+1}[/tex]

can some help me out with this EXTRA POINTS!!

Answers

Answer: ≈ 185.73009

Step-by-step explanation:

area of square: [tex]length^{2}[/tex]

area of circle: [tex]\frac{radius^{2} * \pi }{y}[/tex]

length: 15

radius = diameter/2

radius: 5

area of square - area of circle =  area between square and circle:

[tex]15^{2} - \frac{5^{2} * \pi }{2} = 225 - \frac{25\pi}{2}[/tex] ≈ 185.73009

hope it helps!

Please mark as brainliest.

The sixth-grade students at Madison Middle School had a walkathon to raise money for their

local hospital. Their principal tracked how much money each student raised. This box plot

shows the results.

Money raised ($)

20

40

60

80

100

120

What percent of the students raised more than $40?

%

Answers

50% of the sixth-grade students at Madison Middle School raised more than $40

What is a box plot?

A box plot provides the summary of a data using the five number summary that is minimum, lower quartile, median, upper quartile and maximum.

From the diagram, the median is $40. Hence 50% of the student raised more than $40

Find out more on box plot at: https://brainly.com/question/14277132

The radius of the Earth is approximately 6,370 kilometers. What is the distance in scientific notation?
A.
637 × 10^2 kilometers
B.
6.37 × 10^3 kilometers
C.
637 × 10^1 kilometers
D.
63.7 × 10^2 kilometers

Answers

Answer:

it is D 63.7 × 10`2 I am not sure if it right tho

Hey there!


• QUESTION READS….

“The radius of the Earth is approximately 6,370 kilometers. What is the distance in scientific notation?”

• THE QUESTION ASKED IN EASIER WAYS

“The SPACE of Earth is about 6,370 km (kilometers). What is the length of the the Earth in scientific notation?”


• GETTING TO KNOW SCIENTIFIC NOTATION…..

It’s the writing of a number in a certain term of a product of an thing anywhere between the numbers 1 - 10 and the other thing is at the power of the number 10

• ANSWERING THE QUESTION BY DOING PROCESS OF ELIMINATION….


Option A.
637 km × 10^2 kn

= 637 km × 10 km × 10 km

= 637 km × 100 km

= 63,700 km

Option A. is incorrect because it is too big.


Option B.
6.37 km × 10^3 km
= 6.37 km × 10 km × 10 km × 10 km

= 6.37 km × 100 km × 10 km

= 6.37 km × 1,000 km

= 6,370 km

Option B. could possibly be your answer


Option C.
637 km × 10^1 km

= 637 km × 10 km

= 6,370 km

Option C. could also possibly be your answer.

Option D.
63.7 km × 10^2 km

= 63.7 km × 10 km × 10 km

= 63.7 km × 100 km

= 6,370 km

Option D. could also possibly be your answer along with B - C.

This is a tricky answer because we have MORE THAN one result.

So, we will just ELIMINATE A, C, D to make it easier.


Thus, we will say your result is:

Option B. 6.37 × 10^3 kilometers



Good luck on your assignment & enjoy your day!



~Amphitrite1040:)

A clock cog rotates 150 degrees in 20 minutes. How far will far will the same cog rotate in 70 minutes

Answers

Answer:

525 degrees

Step-by-step explanation:

I think it’s 450
I am not sure but yeah ;)

The volume of a right triangular prism is 72 cubic feet. the height of the prism is 9 feet. the triangular base is an isosceles right triangle. a right triangular prism has a volume of 72 cubic feet. triangles a e c and b f d are the base triangles. sides f d and b f are congruent and form a right angle. the height of the prism is 9 feet. what is the area of the triangular base? square feet what is the length of edge df? feet

Answers

Answer:

8 square feet

Step-by-step explanation:

The formula of a volume of a prism:

We have

Substitute:

            divide both sides by 9

Answer:

8 and 4

Step-by-step explanation:

The first one is 8

The second one is 4

A tank in the shape of a hemisphere has a radius of 7 feet. if the liquid that fills the tank has a density of 93 pounds per cubic foot, what is the total weight of the liquid in the tank, to the nearest full pound?

Answers

The total weight of the liquid that filled the hemispheric tank to the nearest full pounds is 66775 pounds.

What is the volume of an hemisphere?

An hemisphere is half of a sphere. Therefore,

volume = 2 / 3 πr³

where,

r = radius

Therefore,

r = 7 feet

volume = 2 / 3 × 3.14 × 7³

volume = 2 / 3 × 3.14 × 343

volume = 2154.04 / 3

volume = 718.013333333

volume = 718.013

Therefore,

density = mass / volume

93 =  mass / 718.013

mass = 718.013 × 93

mass = 66775.24

mass ≈ 66775 pounds

learn more on hemisphere here: https://brainly.com/question/14527816

1. 4 A box contains 300 discs of different colours. There are 100 pink discs, 100 blue discs and 100 orange discs. The discs of each colour are numbered from 0 to 99. Five discs are selected at random, one at a time, with replacement. Find


(i) the probability that no orange discs are selected. [3]


(ii) the probability that exactly 2 discs with numbers ending in a 6 are selected. [4]


(iii) the probability that exactly 2 orange discs with numbers ending in a 6 are selected. [4]


(iv) the mean and variance of the number of pink discs selected. [8]


1. 5 The mean number of defective batteries in packs of 20 is 1. 6. Use a binomial distribution to calculate the probability that a randomly chosen pack of 20 will have more than 2 defective batteries

Answers

The probability that no orange discs are selected is 0.1295 and the probability that exactly 2 discs with numbers ending in a 6 are selected is 0.0721.

How to calculate probability?

The probability that no orange discs are selected will be:

= 200C5 / 300C5

= 0.1295

The probability that exactly 2 discs with numbers ending in a 6 are selected will be:

= (30C2 × 270C3) / (300C5)

= 0.0721

The probability that exactly 2 orange discs with numbers ending in a 6 are selected will be:

= (100C2 × 290C3) / (300C5)

= 0.0092

The mean of the number of pink discs selected will be:

= np

= 5 × 1/3

= 5/3

The variance will be:

= 5 × 1/3 × 2/3

= 10/9 = 1.11

Learn more about probability on:

https://brainly.com/question/24756209

can somone help? ill give brainlist

Answers

Answer:

13.99+.75+.75=15.49

Part A ) 15.49

Part B) Anna is correct because her pizza only cost 15.49 and if you subtract 15.49 from 20 it is 14.51

Step-by-step explanation:

Find the volume of the prism.

Answers

Answer:   The formula for the volume of a prism is obtained by taking the product of the base area and height of the prism. The volume of a prism is given as V = B × H where, "V" is the volume of the prism, "B" is the base area of the prism, and "H" is the height of the prism.

If a sequence is 0,1,1,2,3,5 what is the 11th number in the sequence?

Answers

Answer:

55

Step-by-step explanation:

This is the Fibonacci sequence! Filling out the rest of the numbers, 55 is the 11th number in this sequence.

oml please don’t say anything that doesn’t help,
do any of you know what 1 3/4 + 1/6 + ____= 7 1/2 is and can you tell me how you got it

Answers

Answer:

first convert mixed fraction to improper fraction then solve by taking LCM

hope the above process helps

Answer:

5  7/12

Step-by-step explanation:

1/6 can be converted to 2/12
1 3/4 can be converted to 1 9/12
Add them together, you'll get 1  11/12
7  1/2 is equal to 7  6/12
7  6/12 - 1  11/12 = 5  7/12

              5 /12 is your answer

Use a ladder method to write 48 and 180 as a product of their prime factors?​

Answers

Factors of 48 are :2,2,2,2,&3. Factors of 180:2,2,3,3,&5.

3x+2x+6v-9g+hd+2v-1g

Answers

Answer:

5x+8x-10g+hd

Step-by-step explanation:

please mark me as brainlest

Hey there!

3x + 2x + 6v - 9g + hd + 2v - 1g

= 3x + 2x + 6v - 9g + 1hd + 2v - 1g

COMBINE the LIKE TERMS

= (3x + 2x) + (6v + 2v) - (9g - 1g) + (1hd)

= 3x + 2x + 6v + 2v - 9g - 1g + 1hd

= 5x + 7v - 10g + 1hd

= 5x + 7v - 10g + hd

= hd - 10g + 8v + 5x


Therefore, your answer is:

hd - 10g + 8v + 5x


Good luck on your assignment & enjoy your day!



~Amphitrite1040:)

A path is made of red tiles and blue tiles.
1/5 of the tiles are red.
a) Write down the ratio of red tiles to blue tiles in its simplest form.

Answers

Answer:

The ratio of red tiles to blue tiles is 1 to 4.

Step-by-step explanation:

Since 1/5 of the tiles are red, 4/5 of the tiles are blue, giving the ratio of red tiles to blue tiles as 1 to 4.

What is the value of x in the figure?


Enter your answer in the box.


x =

Answers

Answer:

68 degrees

Step-by-step explanation:

The straight horizontal line is a straight angle with 180 degrees it is split in a way that you know under the right angle, is another right ange making up part of the 180° line, add 22 and then subtract what you get from 180 to get 68.

A simpler way, add the angles in the X and 22 degree things and there is a right angle, 90 degrees, subtract 22 from 90 and get 68

Two different ways, either will work

Hope this helps!

Angles are complementary

x+22=90x=90-22x=68

x is 68°

5. A rectangle has an area of 16 cm². Do you know its length?

Answers

Answer:

144 [tex]cm^2[/tex]

Step-by-step explanation:

Answer:the answer were to your question is 4

Step-by-step explanation:

Put me on brainlyiest pls

100pts Help rounding!

Answers

Answer:

5

Step-by-step explanation:

5 158/559

= 5 + 158/559

= 5 + 0.28264758

= 5.28264758

= 5 (rounded to nearest whole number)

Answer:

5

Explanation:

[tex]\rightarrow \sf 5\dfrac{158}{559}[/tex]

[tex]\rightarrow \sf 5.282647585[/tex]

The whole number is the quotient in any fraction.

Need help with solving for X

Answers

Answer:

[tex]\huge\boxed{\sf x = 10}[/tex]

Step-by-step explanation:

Since the two angles are complementary angles, the measures of these angles add up to 90 degrees.

5x + 25 + x + 5 = 90

5x + x + 25 + 5 = 90

6x + 30 = 90

Subtract 30 to both sides

6x = 90 - 30

6x = 60

Divide 6 to both sides

x = 60 / 6

x = 10

[tex]\rule[225]{225}{2}[/tex]

Is each statement true or false?
Explain your answers
A.
[tex]m \: = x[/tex]
B.
[tex]z \: = x + y[/tex]
C.
[tex]m \: = y + z[/tex]
D.
[tex]m = 180 - x[/tex]
E.
[tex]x = 180 - (y + z)[/tex]

(the triangle is on the image)




Answers

Answer:

  A, B: False

  C, D, E: True

Step-by-step explanation:

The angle sum theorem and the definition of a linear pair of angles can help you answer these questions.

A.

  m = x; False. Angles m and x are a linear pair. They are supplementary (total 180°). They are only equal if they are both 90°, which is not indicated in this diagram as being the case.

__

B.

  z = x+y; False. Angles x, y, and z total 180° according to the angle sum theorem. The equation is only true if z = 90°, which is not indicated in this diagram as being the case.

__

C.

  m = y +z; True. This is explained by the equations of D and E, which are also both true. The theorem that expresses this relationship is, "an exterior angle is equal to the sum of the remote interior angles."

__

D.

  m = 180° -x; True. This is exactly what we mean when we say angles m and x are a linear pair. Together, they form a straight line. The angles of a linear pair are supplementary, meaning their sum is 180°. Angle m is also called an "external angle" of the triangle.

__

E.

x = 180° -(y +z); True. The angles of a triangle total 180° according to the angle sum theorem. That means the value of any one of them is the difference between 180° and the sum of the other two. This is effectively telling you that angle x is supplementary to the sum of angles y and z.

Above, we noted that angle x is supplementary to angle m. Angles that are supplementary to the same angle have the same measure, so ...

  m = 180° -x

  180° -x = y +z . . . . . rearrange equation E

  m = y +z

Which of these numbers is a solution of the inequality 3x>18?

Answers

hello!

Let's solve this inequality:-

3x<18

Divide both sides by 3:-

x<6

So numbers less than 6 will make this inequality [tex]\sf{true}[/tex].

notes:-

You haven't listed any numbers, so I don't know what options you have,

Please list the numbers in the comments & I will edit my answer :)

Hope everything is clear; if you need any explanation/clarification, kindly let me know, and I'll comment and/or edit my answer :)

Answer:

The possible answers are all numbers above 6.

Step-by-step explanation:

3x>18

3x/3> 18/3

x> 6

1000 masks were distributed equally among a certain number of people. if there were 5 more people more , each would have received 10 masks less. among how many people was the mask distributed ?find it .​

Answers

Answer:

10+5 =15 thanks for asking

Which function has the given properties below? the domain is the set of all real numbers. one x-intercept is (startfraction pi over 2 endfraction, 0 endfraction). the maximum value is 3. the y-intercept is (0, –3). y = –3sin(x) y = –3cos(x) y = 3sin(x) y = 3cos(x)

Answers

The required function with maximum value 3 and x and y-intercepts π/2 and 3 respectively is y=-3cos(x)

What is a function?

A function is a rule that relates two variables.

Let us check the x-intercept, y-intercept, and maximum value for each function.

1)y =-3Sin(x)

for x-intercept put y=0

0 = -3Sin(x)

Sin(x) =0

x=0

for y-intercept put x=0

y=-3Sin0

y=0

The maximum value of the given function =the lower limit of the range of -3Sin(x) i.e. 3

Similarly, for y = -3cos(x)

x-intercept =π/2

y-intercept = 3

Maximum value of function = lower limit  of the range of  3cos(x) i.e. 3

Therefore, the required function with maximum value 3 and x and y-intercepts π/2 and 3 respectively is y=-3cos(x)

To get more about functions visit:

https://brainly.com/question/17043948

Answer:

B.    y = –3cos(x)

Step-by-step explanation:

Im extremely sure, have a great day

Aiden paid 45. 15 to stream three movies. All three movies were the same price. How much did each movie cost

Answers

Answer:

Each movie costs $15.05

Step-by-step explanation:

The price that was paid was $45.15

We know that there were 3 movies brought at the same prices.

We want to know how much each movie costs.

This is a simple division problem.

This is because the keywords "same" and also "each"

Do 45.15 divided by 3

45.15/3 = 15.05

Your answer is $15.05

You could verify by adding $15.05 three times.

$15.05 + $15.05 + $15.05 = $45.15

Therefore, Each movie costs $15.05

Gerard has 365 baseball cards he puts as many of them into piles of 100 how many piles of 100 does he make

Answers

Answer would be 3. 365 / 100 = 3.65, however this is reality math and you cannot have fractional piles. Therefore, 3.

ASAP PLS An angle measures 72.6° less than the measure of its complementary angle. What is the measure of each angle?

Answers

Answer:

Step-by-step explanation:

How can you find percents

Answers

Answer:

To find the percentage of a number when it is in decimal form, you just need to multiply the decimal number by 100. For example, to convert 0.5 to a percentage, 0.5 x 100 = 50%. The second case involves a fraction. If the given number is in fractional form, first convert it to a decimal value and multiply by 100.

Step-by-step explanation:

. For example, to convert 0.5 to a percentage, 0.5 x 100 = 50%

hope this will help

Mandy bought a rectangular shelf that is 1 2/3 feet long and 5/8 feet wide. What is the area of the shelf? Enter your answer as a mixed number in simplest form by filling in the boxes. $$ ft²

Answers

The area of a rectangular shelf with length of 1 2 / 3 ft and width of 5 / 8 ft is  [tex]1\frac{1}{24}[/tex] ft

How to find area of a rectangle?

A rectangle is a quadrilateral. It has 2 opposite sides equal to each other.

Therefore,

area of rectangle = lw

where

l = lengthw = width

Therefore,

length = 1  2 / 3  = 5 / 3 ft

width = 5 / 8 ft

area = 5 / 3 × 5 / 8

area = 25 / 24 ft

area = [tex]1\frac{1}{24}[/tex] ft

learn more on rectangle here: https://brainly.com/question/1106604

Ram is 3 years younger than Shyam and Krishna is 5 years older than Shyam . If the product of percentages of Ram and Krishna is 48, How old is Shyam?​

Answers

Answer:

Shyam would be 43 years old

Step-by-step explanation:

Ram is 49 years old, since she is 3 years younger than Shyam and Krishna is 48 years old than Shyam

Martin has a pencil case which contains 4 blue pens and 3 green pens. Martin picks a pen at random from the pencil case. He notes its colour, and then replaces it. He does this two more times. Work out the probability that when Martin takes three pens, exactly two are the same colour. (3 marks)

Answers

Answer: How do you not understand this?

I understand it perfectly.

Step-by-step explanation:

The  probability that when Martin takes three pens, exactly two are the same color is 26.53%.

Probability

Given:

Blue pens=4

Green pens=3

Hence:

Probability=4³+3³÷(4+3)³

Probability=64+27/343

Probability=91/343

Probability=0.2653×100

Probability=25.53%

Therefore the  probability that when Martin takes three pens, exactly two are the same color is 26.53%.

Learn more about probability here:https://brainly.com/question/24756209

#SPJ6

Other Questions
Please help!! It has to be *TWO* angles {An angle measures 150 less than the measure of its supplementary angle. What is the measure of each angle?} The density of a substance is equal to 2.5 g/mL. The sample you have has a mass of 13g. How much space does this object take up? ______ is recommended for promotion when a high degree of service or support, known as ancillary services, is required after the sale of the product. Did native americans celebrate the winter solstice ? The triangular right prism shown below has a triangular base whose perimeter is 22.4 inches and whose area is 15.2 inches. If its height is 9.7 inches, then which of the following is its volume in cubic inches? Help me pls -3^3(-8n^3-3n^2) 3 8 B. Maria bought 13 pounds of nails for a carpentry project. She ended up with pound of nails when she finished the project. a. Write an equation that can be used to determine how much of the 13 pounds of nails Maria used I'm looking for a equation. There is a proportional relationship between the revenue (in dollars), y,and the number of whistles Glados sells, x. She sells 98 whistles for$980.Graph this proportional relationship. What is the area and perimeter for this Consider the reaction.A(aq)2B(aq)=4.87106 at 500 KIf a 2.00 M sample of A is heated to 500 K, what is the concentration of B at equilibrium? HEEEELPPPP pleaseeee????? Which line from the text is the best evidence that the US and Britain supported colonial independence? representing His Majestys Government in the United Kingdomdeem it right to make known certain common principlesbase their hopes for a better future for the worldwish to see sovereign rights and self government restoredThe President of the United States of America and the Prime Minister, Mr. Churchill, representing His Majesty's Government in the United Kingdom, being met together, deem it right to make known certain common principles in the national policies of their respective countries on which they base their hopes for a better future for the world. . . .Third, they respect the right of all peoples to choose the form of government under which they will live; and they wish to see sovereign rights and self government restored to those who have been forcibly deprived of them.The Atlantic Charter, 1941 A sum amounts Rs. 720 is divided into two parts in the ratio 3:5. How much will be the amount in second part?) Ans: Rs. 450 where is A what is the answer? Which of the following are examples of habitats where sympatric speciation could occur?Choose one or more:A. different locations along the Great Barrier coral reef in AustraliaB. opposite sides of a mountain rangeC. the North Rim and South Rim of the Grand CanyonD. an elevational gradient on the south-facing slope of a mountainE. varying water depths in a lake Colin & Brian share a lottery win of 2100 in the ratio 4 : 1.Colin then shares his part between himself, his wife & their son in the ratio 2 : 5 : 3.How much more does his wife get over their son? Which structure determines the traits that will be passed to offspring?. Introduction to Functions (40-41)Find which relations below represent functions. Write function if it represents a function in the blanks provided.Otherwise, write not a function in the blanks provided below the tables.ABX1YX1X1y703-4-3557-41-827910-4-1-1157-4-37-2or do2057-4-51 Which expression is equivalent to 7(4x + 2y)?A) 42xyB) 28x + 2yC) 11x + 2yD) 28x + 14y What is the significance of the motto In God We Trust? What issue were modern conservatives concerned about in the 1980s?A. The Vietnam WarB. Health careC. AbortionD. Space exploration